Questions   
Sort: 
 #3
avatar+26367 
+2

Find all real x that satisfy (2^x - 4)^3 + (4^x - 2)^3 = (4^x + 2^x - 6)^3

 

\(\begin{array}{|lrcll|} \hline (2^x - 4)^3 + (4^x - 2)^3 = (4^x + 2^x - 6)^3 \\ \qquad\text{We substitute:} ~ a = 2^x,~ b = 4^x,~ c = -4,~ d = -2 \\ (a +c)^3 + (b +d)^3 = (a+b+c+d)^3 \\ \hline \end{array}\\ \small{ \begin{array}{|lrcll|} \hline a^3+3ac(a+c)+c^3+b^3+3bd(b+d)+d^3 & =& a^3 + b^3 + c^3+ d^3 \\ && + 6 a b c + 6 a b d + 6 a c d + 6 b c d \\ & & + 3 a^2 b + 3 a^2 c + 3 a^2 d + 3 a b^2 \\ & & + 3 a c^2 + 3 a d^2 + 3 b^2 c + 3 b^2 d \\ & & + 3 b c^2 + 3 b d^2 + 3 c^2 d + 3 c d^2 \\ \not{a^3}+\not{b^3}+\not{c^3}+\not{d^3}+\not{3a^2c}+\not{3ac^2}+\not{3b^2d}+\not{3bd^2} & =& \not{a^3} + \not{b^3} + \not{c^3}+ \not{d^3} \\ && + 6 a b c + 6 a b d + 6 a c d + 6 b c d \\ & & + 3 a^2 b + \not{3 a^2 c} + 3 a^2 d + 3 a b^2 \\ & & + \not{3 a c^2} + 3 a d^2 + 3 b^2 c + \not{3 b^2 d} \\ & & + 3 b c^2 + \not{3 b d^2} + 3 c^2 d + 3 c d^2 \\ \hline \end{array}}\\ \small{ \begin{array}{|rcll|} \hline 0 &=& 6 a b c + 6 a b d + 6 a c d + 6 b c d + 3 a^2 b + 3 a^2 d + 3 a b^2 + 3 a d^2 + 3 b^2 c + 3 b c^2 + 3 c^2 d + 3 c d^2 ~ | ~: 3 \\ 0 &=& 2 a b c + 2 a b d + 2 a c d + 2 b c d + a^2 b + a^2 d + a b^2 + a d^2 + b^2 c + b c^2 + c^2 d + c d^2 \\ 0 &=& a^2(b+d)+b^2(a+c)+c^2(b+d)+d^2(a+c)+2bd(a+c)+2ac(b+d) \\ 0 &=& (b+d)(a^2+2ac+c^2)+(a+c)(b^2+bd+d^2) \\ 0 &=& (b+d)(a+c)^2+(a+c)(b+d)^2 \\ 0 &=& (b+d)(a+c)(a+b+c+d) \\ \hline \end{array}}\)

 

Solution:

\(\begin{array}{|rcll|} \hline \mathbf{b+d} &\mathbf{=}& \mathbf{0} \\ 4^x-2 &=& 0 \\ 4^x &=& 2 \\ 2^{2x} &=& 2^1 \\ 2x &=& 1 \\ \mathbf{x_1} &\mathbf{=}& \mathbf{\dfrac{1}{2}} \\ \hline \end{array}\)

 

\(\begin{array}{|rcll|} \hline \mathbf{a+c} &\mathbf{=}& \mathbf{0} \\ 2^x-4 &=& 0 \\ 2^x &=& 4 \\ 2^{x} &=& 2^2 \\ x &=& 2 \\ \mathbf{x_2} &\mathbf{=}& \mathbf{2} \\ \hline \end{array}\)

 

\(\begin{array}{|rcll|} \hline \mathbf{a+b+c+d} &\mathbf{=}& \mathbf{0} \\ 2^x + 4^x-6 &=& 0 \\ (2^x-2)(2^x+3) &=& 0 \\ \hline 2^x-2 &=& 0 \\ 2^x &=& 2^1 \\ x &=& 1 \\ \mathbf{x_3} &\mathbf{=}& \mathbf{1} \\\\ 2^x+3 &=& 0 \\ 2^x &=& -3 \\ x\log(2) &=& \log(-3) \quad & |\quad \text{ no solution, $log(-3)$ complex } \\ \hline \end{array}\)

 

laugh

Jan 25, 2018
 #1
avatar+26367 
+2

Find an ordered triple (x,y,z) of real numbers satisfying x<= y<= z and the system of equations
sqrtx + sqrty +sqrtz = 10
x+y+z=38
(sqrtxyz) = 30

 

\(\begin{array}{|lrcll|} \hline (1) & \sqrt{x} + \sqrt{y} +\sqrt{z} &=& 10 \\ (2) & x+y+z &=& 38 \\ (3) & \sqrt{xyz} &=& 30 \\ \hline \end{array}\)

 

1.

\(\begin{array}{|lrcll|} \hline & (\sqrt{x} + \sqrt{y} +\sqrt{z})^2 &=& 10^2 \\ & \underbrace{x+y+z}_{=38} +2( \sqrt{xy}+\sqrt{xz}+\sqrt{yz} ) &=& 100 \\ & 38 +2( \sqrt{xy}+\sqrt{xz}+\sqrt{yz} ) &=& 100 \quad & | \quad -38 \\ & 2( \sqrt{xy}+\sqrt{xz}+\sqrt{yz} ) &=& 62 \quad & | \quad :2 \\ \mathbf{(4)}& \mathbf{\sqrt{xy}+\sqrt{xz}+\sqrt{yz}} &\mathbf{=}& \mathbf{31} \\ \hline \end{array}\)

 

\(\begin{array}{lcll} \text{We substitute:} \\ \quad x_1 = \sqrt{x} & \text{ or }& x = x_1^2 \\ \quad x_2 = \sqrt{y} & \text{ or }& y = x_2^2 \\ \quad x_3 = \sqrt{z} & \text{ or }& z = x_3^2 \\ \end{array}\)

 

\(\begin{array}{|rcll|} \hline x_1+x_2+x_3 &=& \sqrt{x} + \sqrt{y} +\sqrt{z} \\ &=& 10 \\ x_1\cdot x_2 \cdot x_3 &=& \sqrt{xyz} \\ &=& 30 \\ x_1\cdot x_2 + x_1 \cdot x_3 + x_2 \cdot x_3 &=& \sqrt{xy}+\sqrt{xz}+\sqrt{yz} \\ &=& 31 \\ \hline \end{array}\)

 

 

\(\begin{array}{lcll} \text{We set:} \\ \quad p &=& -(x_1+x_2+x_3) \\ &=& -10 \\ \quad q &=& x_1\cdot x_2 + x_1 \cdot x_3 + x_2 \cdot x_3 \\ &=& 31 \\ \quad r &=& -(x_1\cdot x_2 \cdot x_3) \\ &=& -30 \\ \end{array}\)

 

\(\begin{array}{lrcll} \text{The cubic equation:} \\ \boxed{ x^3 - 10x^2+31x-30 = 0 } \\ \end{array}\)

 

The first solution is \(x_1 = 2\)

 

Long division:

\(\begin{array}{rcll} x^2-8x+15 = (x-3)(x-5) \end{array}\)

so \(x_2 = 3\) and \(x_3 = 5\)

 

\(\begin{array}{|rcll|} \hline x_1 = 2 & x &=& x_1^2 \\ & \mathbf{x} &\mathbf{=}& \mathbf{4} \\\\ x_2 = 3 & y &=& x_2^2 \\ & \mathbf{y} &\mathbf{=}& \mathbf{9} \\\\ x_3 = 5 & z &=& x_3^2 \\ & \mathbf{z} &\mathbf{=}& \mathbf{25} \\ \hline \end{array}\)

 

The ordered triple (4, 9, 25) of real numbers satisfying \(4\le 9\le 25\) and the system of equations.

 

laugh

Jan 25, 2018
 #1
avatar+26367 
+3

Find a polynomial $f(x)$ of degree $5$ such that both of these properties hold:
$f(x)-1$ is divisible by $(x-1)^3$.
$f(x)$ is divisible by $x^3$.


1. \(\bf{\text{ $\mathbf{f(x)}$ is divisible by $\mathbf{x^3}~$ ?}}\)

 

\(\begin{array}{lcll} \text{Let $f(x) = a(x-x_1)(x-x_2)(x-x_3)(x-x_4)(x-x_5)$ is a polynomial of degree $5$.} \\ \text{If f(x) is divisible by $x^3$, than $x_1=x_2=x_3=0 $} \\ \qquad f(x) = a(x-0)(x-0)(x-0)(x-x_4)(x-x_5) \\ \qquad \boxed{f(x) =ax^3(x-x_4)(x-x_5)} \text{ This polynom is divisible by $x^3$} \\ \text{expand to:} \\ \qquad f(x)= ax^5 - a(x_4+x_5)x^4 + ax_4x_5x^3 \qquad \text{Let $A=a$, $~B=a(x_4+x_5)$, and $~C = ax_4x_5 $ } \\ \text{finally we have:} \\ \qquad \boxed{f(x) = Ax^5+Bx^4+Cx^3 \qquad (1) } \text{ This polynom is divisible by $x^3$} \\ \end{array}\)

 

2. \(\bf{\text{ $\mathbf{f(x)-1}$ is divisible by $\mathbf{(x-1)^3}~$ ?}}\)

 

\(\begin{array}{lcll} \text{Say $P(x) $ is a polynom divisible by $(x-1)^3$ .} \\ \text{Set:}\\ \qquad P(x)=b\cdot(x-1)^3 \\ \qquad \text{The root is $x = 1$ }\\ \text{so}\\ \qquad P(1)=b(1-1)^3=0 \\ \text{Set also:}\\ \qquad \text{ $P'(x)= 3b(x-1)^2 $}\\ \text{so}\\ \qquad P'(1)= 3b(1-1)^2=0 \\ \text{Set finally:}\\ \qquad \text{ $P''(x)= 6b(x-1) $}\\ \text{so}\\ \qquad P''(1)= 6b(1-1)^2=0 \\\\ \text{Conclusion} \\ \text{If $P(x)$ is divisible by $(x-1)^3$, so $\boxed{\mathbf{P(1)=P'(1)=P''(1)=0}\qquad (2)}$ at the root $x=1$ } \\ \end{array}\)

 

\(\begin{array}{lrcll} \text{We set:}\\ \text{1)}& f(x) - 1 &=& P(x) \quad & | \quad \text{$f(x)-1$ and $P(x)$ are divisible by $(x-1)^3 $ }\\ & f(1) -1 &=& P(1) = 0 \quad & | \quad \rightarrow (2) \\\\ \text{2)}& (f(x) - 1)' &=& P'(x) \\ & f'(x) &=& P'(x) \\ & f'(1) &=& P'(1) = 0 \quad & | \quad \rightarrow (2) \\\\ \text{3)}& (f(x) - 1)'' &=& P''(x) \\ & f''(x) &=& P''(x) \\ & f''(1) &=& P''(1) = 0 \quad & | \quad \rightarrow (2) \\ \end{array}\)

 

\(\begin{array}{lrcll} \text{We see:}\\ & \boxed{ \begin{array}{r} f(1) -1 = 0 \qquad (3) \\ f'(1) = 0 \qquad (4) \\ f''(1)= 0 \qquad (5) \\ \end{array} } \end{array}\)

 

\(\begin{array}{lrcll} \text{We calculate:}\\ & f(x) &=& Ax^5+Bx^4+Cx^3 \\ & f(x) -1 &=& Ax^5+Bx^4+Cx^3 - 1 \\ & f(1)-1 &=& A+B+C -1 = 0 \quad & | \quad \rightarrow (3) \\\\ & f'(x) &=& 5Ax^4+4Bx^3+3Cx^2 \\ & f'(1) &=& 5A+4B+3C = 0 \quad & | \quad \rightarrow (4) \\\\ & f''(x) &=& 20Ax^3+12Bx^2+6Cx \\ & f''(1) &=& 20A+12B+6C = 0 \quad & | \quad \rightarrow (5) \\ \end{array}\)

 

\(\begin{array}{lrcll} \text{Solve the Simultaneous Equations, we calculate $A~$, $B~$, $C~$ :} \\ \end{array}\\ \begin{array}{lrcll} A+B+C -1 = 0 \quad \Rightarrow & A+B+C &=& 1 \\ & 5A+4B+3C &=& 0 \\ & 20A+12B+6C &=& 0 \\ \end{array}\)

 

Cramer's Rule:

\(\begin{array}{rcrcrcr} 1\cdot A &+& 1\cdot B &+& 1\cdot C &=& 1 \\ 5\cdot A &+& 4\cdot B &+& 3\cdot C &=& 0 \\ 20\cdot A &+& 12\cdot B &+& 6\cdot C &=& 0 \\ \end{array}\\ \small{ \begin{array}{lcl} \\ \text{Determinant denominator} &=& \begin{vmatrix} 1&1&1 \\ 5&4&3 \\ 20&12&6 \\ \end{vmatrix}\\ \\ &=& 1\cdot 4\cdot 6 + 5\cdot 12\cdot 1 +20\cdot 1\cdot 3 - 20\cdot 4\cdot 1 -1\cdot 12\cdot 3 -5\cdot 1\cdot 6\\ &=& 24+60+60-80-36-30 \\ &=& -2 \\ \end{array} } \)

 

\(\begin{array}{lcl} A &=& \dfrac{ \begin{vmatrix} 1&1&1 \\ 0&4&3 \\ 0&12&6 \\ \end{vmatrix} }{-2}\\\\ &=&\dfrac{ 1\cdot 4\cdot 6 - 1\cdot 12\cdot 3 } {-2}\\ &=&\dfrac{ -12 } {-2}\\\\ \mathbf{A} & \mathbf{=} & \mathbf{6}\\ \end{array} \begin{array}{lcl} B &=& \dfrac{ \begin{vmatrix} 1&1&1 \\ 5&0&3 \\ 20&0&6 \\ \end{vmatrix} }{-2}\\\\ &=&\dfrac{ 20\cdot 1\cdot 3 - 5\cdot 1\cdot 6 } {-2}\\ &=&\dfrac{ 30 } {-2}\\\\ \mathbf{B} & \mathbf{=} & \mathbf{-15}\\ \end{array} \begin{array}{lcl} C &=& \dfrac{ \begin{vmatrix} 1&1&1 \\ 5&4&0 \\ 20&12&0 \\ \end{vmatrix} }{-2}\\\\ &=&\dfrac{ 5\cdot 12\cdot 1 - 20\cdot 4\cdot 1 } {-2}\\ &=&\dfrac{ -20 } {-2}\\\\ \mathbf{C} & \mathbf{=} & \mathbf{10}\\ \end{array}\)

 

\(\begin{array}{lrcll} \text{The Polynom $f(x) = Ax^5+Bx^4+Cx^3$ with $A=6$, $~B=-15$, and $~C = 10$}\\ \quad \boxed{f(x)=6x^5-15x^4+10x^3} \end{array}\)

 

Proof:

\(\begin{array}{|rcll|} \hline f(3) &=& 6\cdot 3^5-15\cdot 3^4 + 10 \cdot 3^3 \\ &=& 1458-1215+270 \\ &=& 513 \\ \frac{f(3)}{3^3} &=& \frac{513}{3^3} \\ &=& 19\ \checkmark \\ \frac{f(3)-1}{(3-1)^3} &=& \frac{513-1}{2^3} \\ &=& \frac{512}{8} \\ &=& 64\ \checkmark \\ \hline \end{array}\)

 

 

laugh

Jan 25, 2018

2 Online Users

avatar
avatar